MEMO 2011 ekipno problem 3


Kvaliteta:
  Avg: 3,0
Težina:
  Avg: 6,0
Dodao/la: arhiva
28. travnja 2012.
LaTeX PDF
For an integer n \geq 3, let \mathcal M be the set \{(x, y) | x, y \in \mathbb Z, 1 \leq  x \leq  n, 1 \leq  y \leq  n\} of points in the plane.

What is the maximum possible number of points in a subset S \subseteq \mathcal M which does not contain three distinct points being the vertices of a right triangle?
Izvor: Srednjoeuropska matematička olimpijada 2011, ekipno natjecanje, problem 3